« first day (1513 days earlier)      last day (3515 days later) » 

1:12 AM
by the way @Ted, it's good I studied that $\omega \wedge d\omega$ problem... since it showed up today
 
The results aren't encouraging, @Studentmath, but I'll finish tomorrow. Hi @Alex @anon
 
LOL
Nice timing
 
You mean the one you got mad at me about, @Mike?
 
No, @Ted, I got mad at myself about that one
 
Sometimes I can't tell the difference.
 
1:13 AM
You'd be hardpressed to find a time I truly got mad at you, I think. I hope.
 
Luckily, I am forgetful.
 
Anyway, there was a question saying "Show that $\omega$ a 1-form is locally of the form $\lambda df$ for some smooth maps to $\Bbb R$ $\lambda, f$ iff $\omega \wedge d\omega = 0$."
 
That's an exercise next spring in my course.
 
It's a nice one.
 
It is just a line of high school algebra :)
 
1:18 AM
Mm? Is it easier than talking about distributions?
 
Yessssss!
 
oh
well, what I said was still valid... so.
The iff is that easy? Not just one direction?
 
Ah, I only assign only if ... If takes Frobenius in some form.
 
Oh, well yes, that part was easy.
 
Yeah, you be right.
 
1:20 AM
Then it sounds like we have the same argument in mind
 
I told you I was forgetful; I'm also senile.
 
Ah, we're the same.
 
1:52 AM
@MikeMiller You're too young to be dead senile. No. No. Please no. snif
 
2:18 AM
@anon you can take a person to be something with an enterance and exit time
to a particular room
those would be randomly distributed (for now, no distribution is assumed)
 
is there only one person in this model? if not, which person is w(t) talking about?
 
Hello @Alexander @anon
 
hi
 
@anon w(t) would be talking about the average of the time that each person so far has spent in the room
 
Hi @robjohn
 
2:20 AM
so if i come in at t = 0 and leave at t = 2 and you come in at t = 1 and leave at t = 2 then we've got an average of 1.5 for w(t)
 
okay, so that's what you mean
 
anyone got a cool algebra problem?
 
i think that we can say that that is $\frac{1}{t}\int_{0}^t N_i(s)ds$ with $N_i(t)$ being the number of people in room $i$ at time $t$
 
@BalarkaSen sure
 
go on
 
2:23 AM
you never said you wanted it
 
($N(t)$ would be integer valued, and not continuous, but still Riemann integrable I'm pretty sure.)
 
bangs head
I already said that I know the solution
And in any case it's not algebra
@MikeMiller =( no problem for me? sirius?
 
@IceBoy hey there
 
@BalarkaSen Let $R$ be a UFD such that all chains of prime ideals $\mathfrak p_0 \subsetneq \mathfrak p_1 \subsetneq \dots \subsetneq \mathfrak p_n$ have $n \leq 2$ (i.e., $R$ has dimension 2). then for any non-unit $\pi \in R$, $R[1/\pi]$ is a PID.
 
Oho.
 
2:27 AM
@robjohn how are your bunnies doing?
 
@MikeMiller Is that supposed to be hard?
 
I make no presumptions about its difficulty
 
You mean you haven't tried it?
 
I've done the problem.
 
Ah, OK.
Well, I'll do it later =P Noted down.
Thinking would continue in January 2016.
 
2:31 AM
bah
if you're not gonna think I'm not gonna provide
 
I will think.
 
okie
 
I was just asking about difficulty because the last problem you gave took me about half a week
 
@IceBoy very well. We just found out they are both male. That is good; we won't end up with a bunny farm ;-)
 
2:33 AM
better than funny barm
wait that went as a failed pun, didn't it?
 
@robjohn good, I was wondering about that to.
 
@TedShifrin hi!
@BalarkaSen hi
 
hello
 
@Alexander Pedro-style : HERRO
 
How are you two
 
2:47 AM
Fine.
 
(alex gruber style: unpunctuated, with no pings)
 
i am ok
 
Twiddling with commutative algebra.
 
(also known as "inept")
 
haha @Alexander
 
2:48 AM
@MikeMiller how is california this time of year
 
quite nice
a few weeks ago it was grody
 
and we are dying with the frequent heavy rains
water water everywhere
 
@MikeMiller I bought a new pair of sunglasses the other day
 
Oh, @AlexanderGruber, have you seen this?
 
with which i have achieved victory in my war with the sky
 
2:50 AM
did you just link a chat message with a link to another chat message
@AlexanderGruber ah; my solution is to close my eyes
 
moi? nope.
 
@BalarkaSen Hahaha well, that's rather consistent
@MikeMiller he sure did
 
or right, yeah. i thought i linked to the search result directly
 
I've mastered the art of walking with my eyes closed in places I know the topology of
 
@MikeMiller Topology of places?
 
Everything is flat then!
 
those are awesome
@BalarkaSen if you think that you've never seen a building
 
well, topologically speaking everything contracts to a point
 
@BalarkaSen Not me.
 
no it certainly does not
 
2:53 AM
@AlexanderGruber You torus?
 
plus, homeomorphism is not homotopy equivalence
get outta here
 
droops ears leaves chat
 
@BalarkaSen you can stay if you can solve a galois theory problem for me
 
I posed it to somebody on facebook, so if you solve it before he does you win
let $L/\Bbb Q$ be Galois, and $K \subset L$ be the subfield generated by all roots of unity in $L$. Suppose that $L = \Bbb Q[a]$, where $a^n \in \Bbb Q$ for some positive integer $n$. Show that $\text{Gal}(L/K)$ is cyclic
 
2:57 AM
Hmm. Thinking out loud : 1 --> Gal(K/Q) --> Gal(L/Q) --> Gal(L/K) --> 1 is short exact, Gal(L/Q) is cyclic.
heh?
 
I've deleted anything that might be seen as a hint
 
oh right cyclic group quotiented are cyclic
 
also, your arrows are pointing in the wrong direction
 
heh?
 
in your sequence
 
3:00 AM
oh yes right
well, i am thinking in the right direction
so it doesn't matter
1 --> Gal(L/K) --> Gal(L/Q) --> Gal(K/Q) --> 1
 
aye
 
You mean $L=\Bbb Q(a)$, surely?
both are the same in any case but standard notations ... =)
 
think about that
 
grabs a cup of ffee coffee
@MikeMiller what d'you mean? $a$ is an algebraic so surely $\Bbb Q [a] \cong \Bbb Q(a)$?
 
right
that's why it doesn't matter how I wrote $L$
 
3:06 AM
i said standard notations sigh
you are a professional expert in wasting times
 
I'm not the one nitpicking identical notation
 
Hello, I have a bold question: when a function is said to be Ck, it means that its first k partial derivatives exists and are continuous, does this means that even the crossed term partial derivatives are continuous? Or does the concept is only valid for derivatives of the same variable?
 
@WeierstraßRamirez "A function" is very broad.
 
and "Pedro" is very bold :D
 
Actually I have very thick blonde hair.
 
3:13 AM
Just had the doubt... I know its completely dull, but if you guys have answer to it I'd be great.
 
Again, do you mean $f:\Bbb R^n\to\Bbb R^m$?
 
Hi @Pedro
All mixed partials of order up to $k$, @WeierstraßRamirez
 
@TedShifrin How's it going?
I wonder what's the weather now around there.
 
Let me make sure if what I'm think is right or not
 
3:17 AM
Super depressed ar how my probability exam is turning out ... Retirement looks certain.
 
@MikeMiller What d'you mean by "$K \subset L$ is generated by all roots of unity in $L$"?
 
@TedShifrin =/ Did it go really bad?
 
@BalarkaSen I mean precisely that.
 
Do you mean $K$ is generated over $\Bbb Q$ by all roots of unity that is in $L$?
 
Getting chillier, at last. I don't want to get sick.
 
3:18 AM
@BalarkaSen He means $K\subset L$ is generated by all roots of unity in $L$.
 
Yes.
 
Cool.
@Pedro I figured that.
 
Not done grading yet, @Pedro, but yes. A few did fine.
 
Well, then $Gal(L/K)$ fixes all the roots of unity in $L$. And the $\alpha$ by which $L$ is generated over $K$ is moved up to $\zeta^k \alpha$ by elts of $Gal(L/K)$
 
@TedShifrin Probability is very counterintuitive!
@FernandoMartin Harro fmartin
 
3:19 AM
hey hey hey
 
hello fernando
 
Awaits for deep question on Galois stuff
 
hi pal
 
@FernandoMartin tell me how to prove Artin's lemma
fast
 
Some of them have no idea what's going on with basic arguments that I've taught numerous times. :(
 
3:20 AM
OH.
 
@MikeMiller: grab a copy of Lang
 
@MikeMiller what lemma?
 
that should do the trick
 
boo
 
what does it state?
 
3:20 AM
hi @Fernando
 
@FernandoMartin supahhotfire.gif
 
Hi @Ted
 
@BalarkaSen one problem at a time, and you've got two
 
OK, OK
 
long time no see
 
3:22 AM
So the action of $Gal(L/K)$ moves $\alpha$ to $\zeta \alpha, \zeta^2 \alpha, \zeta^3 \alpha$ and whatnots. Hmm.
$\prod_k \left ( x - \zeta^k \alpha \right )$ is left invariant under the action.
\O_o/ I am just messing about, aren't I?
 
well, yes, but how else does one solve problems
 
let $\zeta$ be the dratted $n$-th root of unity. then the invariant polynomial is $x^n - \alpha^n$.
erm but that doesn't help.
meh i think i'll give up this approach
Wohahwhawhawha wait
@MikeMiller Let $n$ be the integer for which $\alpha^n \in \Bbb Q$. Then $x^n - \alpha^n \in \Bbb Q[x]$. And the splitting field is precisely $L/\Bbb Q$.
 
right
 
Then $Gal(L/\Bbb Q)$ is cyclic, right?
 
3:34 AM
say what
 
probably not
if it is, you'll have to prove it
I don't have a counterexample in my hat but there's surely one
 
i dun believe you.
 
if you believe it's true, prove it
 
oh noes it's false.
take $x^4 - 2$ over $\Bbb Q$. Galois group is dihedral.
in fact take $x^3 - 2$ =P
 
woo
 
3:40 AM
it startes to look hopeless to moi. i'd better get some sleep.
 
later pal
 
i'm still right here
 
don't you want to get some sleep?
 
i do, but i am not going to
 
3:54 AM
@MikeMiller $Gal(L/K)$ just acts on $\alpha$ by moving it to $\zeta^k \alpha$, whereas all $\zeta$s are left fixed. so elts $a + b\alpha$ of $K(\alpha)$ are moved to $a + b \zeta^k \alpha$. $x \mapsto x\zeta$ is a generator for these, right?
 
what's x
 
i meant $\alpha$
 
no reason that map in general should actually define an automorphism
or that it should fix $\zeta$s
 
hmm right
humph
 
4:27 AM
@MikeMiller Are you there?
 
sorta
 
Well as $L/\Bbb Q$ is galois, and as $\alpha \in L$, the $n$-th root of unity is in $L$ automatically, right?
So $L \supseteq \Bbb Q(\zeta_n)$.
you following, @MikeM?
 
I can't right now, I'm sorry
I shoulda said no :P
Talk later
 
Byes.
sigh
 
 
2 hours later…
6:24 AM
There was a recent discussion on meta: Are congratulary post off-topic on meta? Or only some of them?. As an experiment, I have created a separate chatroom which could (possibly) serve as a place for such posts; perhaps more suitable than meta.

 In praise of Math.SE site and its users

achievements, milestones, interesting statistics
 
6:35 AM
thanks martin
 
Maybe you could add Wikipedia link or some short explanation what smooth number is to your question.
Admittedly, people who do not know this notion (like me) will probably not be able to answer your question. But the question will be more self contained and you will help a few people to learn something new.
 
Greetings
@robjohn have you seen Kirill's answer? Crazy awesome ...
 
@DanBrumleve Maybe you can also mention your question in number theory chatroom. There are not many people who visit it, but those who do might be interested in the question.
 
@Chris'ssis Did you have any progress on the Cleo Fey situation?
 
@Chris'ssis To what? I am just about to finish my asymptotic expansion.
 
6:44 AM
2
A: Computing $\lim\limits_{n\to\infty} \Big(\sum\limits_{i = 1}^n \sum\limits_{j = 1}^n \frac1{i^2+j^2}-\frac{\pi}{2} \log(n)\Big)$.

KirillThe limit in question is equal to $$\def\tfrac#1#2{{\textstyle\frac{#1}{#2}}} \tfrac14\pi\log\left(\frac{16\pi^3e^{2\gamma}}{\Gamma(\frac14)^4}\right) -G-\zeta(2) \\ = -0.82586\ 11759\ 78831\ 08201\ 02008\ 35613\ 80953\ 63017\ 94512\ 34066\ 96955\ 08772 \ldots $$ in terms of Euler's $\gamma$ and ...

 
Yo.
 
@Committingtoaname These days I'm busy with some other kind of stuff ... no time for that ...
 
added wiki link for smooth
i will check out number theory room
also i had this math.stackexchange.com/questions/943964/… which is sort of the motivation for the other question
 
@Chris'ssis Having that formula with the Sierpinski constant at hand makes it pretty easy. Showing that would be interesting, but I guess we can take it for granted. However, I like to understand all steps. I will finish my answer with the asymptotic expansion anyway.
 
@robjohn I wasn't aware of that formula :-(
 
6:52 AM
@Chris'ssis I wasn't either. I would like to see a derivation of it.
 
@robjohn Me too!
 
@Chris'ssis when I finish the asymptotic expansion (which is interesting in the fact that I use some divergent series), I can verify that numerical answer :-)
 
OK :-)
 
I use the divergent series $$\sum_{j=0}^\infty(-1)^j\binom{2j}{n} =\mathrm{Re}\left[\frac{i^n}{(1-i)^{n+1}}\right]$$
which can be written as a finite sum of binomial coefficients
 
hmmm, interesting
 
6:58 AM
You multiply each term by $\alpha^{2j}$, so that the series converges, then let $\alpha\to1$
This allows us to get an asymptotic series for $$\sum_{k=1}^n\frac1{n^2+\alpha^2j^2}$$
then let $\alpha\to1$
 
Why do we require the base elements for a product space to consist of all but finitely copies of the entire spaces?
@MikeMiller Oh hey.
 
hello
 
7:14 AM
@robjohn Then it seems that if we have a triple sum, we should consider a sphere sum, shouldn't we? (referring to the Kirill's answer)
 
@Chris'ssis I am not sure I follow you...
 
(this is only if out there is an asymptotic for such a sum)
@robjohn In 2 dimensions we have square and circle for the distribution of the numbers , right? In 3 dimensions we have the cube and the sphere. (at least I mean, but this might be convenient for us)
 
@Chris'ssis Oh, this is just an extension of the problem, not dealing at all with my asymptotic expansion?
 
@robjohn Exactly. :-)
 
@Chris'ssis Oh, sorry... I am concentrating on finishing my answer, so I wasn't going that way
 
7:26 AM
@robjohn OK. I'm very curious about what you finally get. Don't hurry. :-)
 
7:37 AM
math.stackexchange.com/questions/945378/what-is-an-axiom-schema other answerer deserves an upvote i think but i am conflicted about it
 
 
1 hour later…
8:40 AM
@robjohn another solution in place
0
A: Computing $\lim\limits_{n\to\infty} \Big(\sum\limits_{i = 1}^n \sum\limits_{j = 1}^n \frac1{i^2+j^2}-\frac{\pi}{2} \log(n)\Big)$.

karvensI happened to struggle on the same problem 3 years ago. Here's another approach. Start from $$\sum_{n=1}^\infty\frac1{x^2+n^2}=-\frac1{2x^2}+\frac{\pi}{2x}+\frac{\pi}{x(e^{2\pi x}-1)}$$ From this we have \begin{array} 1\sum_{m\le N}\sum_{n\le N}\frac{1}{m^2+n^2}&=\sum_{m\le N}\left(-\frac{1}{2m^...

I'm amazed of what happens there. Niceeeeeeeeeeeeeeeeeeeeeeee!!!
 
9:05 AM
@robjohn The last answer sent me to the recovery room, I need some time to recover myself ... that one was a bomb of beauty ...
 
9:42 AM
@Chris'ssis I'm going to have to look at those. I am still transferring my series to LaTeX. I should be finished in a bit and verify the answers numerically.
 
@robjohn OK
bbl
 
10:05 AM
Does anyone here know how to use Mathematica?
 
@Chris'ssis I am now computing the sum for $n=10000$ to 80 places so that I can use it to compute the constant to 72 places
 
Hello...
Knock, knock...
 
Not well @Anastasiya
 
10:21 AM
@Committingtoaname Do you know how to use Mathematica?
 
Very barely
@Anastasiya-Romanova I could try to help you
I can do generating functions, plotting, sums, simplification, integrals and some other stuff
 
Does Mathematica always work online? I mean we must connect to internet to be able to run it
 
I have never tried to be honest, but I doubt it. You would lose Wolframalpha utility, but otherwise it should work
 
When I turn off my internet connection, I can't run Mathematica.
No, it doesn't work
 
I'll turn my internet off to try it and come back in a min
 
10:27 AM
OK
 
It gave me a warning that no connection was detected, and I clicked Create new 'notebook', and everything seemed fine
 
@Committingtoaname It still doesn't work. Here is the output when I work offline: FetchURL::offline: Mathematica is currently configured not to use the Internet. To allow Internet use, check the "Allow Mathematica to use the Internet" box in the Help [FilledRightTriangle] Internet Connectivity dialog.
I want to use Mathematica in offline mode if it's possible
 
Huy
@Anastasiya-Romanova: You bought Mathematica without knowing whether it's possible to use it offline in advance?
Or maybe the trial version is limited?
 
No, I don't know that
 
Huy
It works perfectly offline for me.
 
10:35 AM
Could you please tell me how to do that?
 
Huy
I just started it, no magic involved.
 
@Huy Try click Help -> Internet Connectivity and un-check Allow Mathematica to access the internet
Does it still work?
 
Huy
I just shut down my laptop and have to do some other stuff before I'm leaving for university, sorry. If you're here in the evening and still have the problem I can try to help.
 
@PedroTamaroff. Your stared comment made me LOL ! ^_^ !!
 
You have a legal copy @Anastasiya-Romanova?
 
10:40 AM
@Committingtoaname Yep!
@Huy OK, I'll wait
 
What version and type of license?
 
ooooooooo
 
@Committingtoaname 9.0.0.0
 
Huy
@Anastasiya-Romanova: Don't you have access to the newest?
 
@Committingtoaname How can I see the type of license?
@Huy I don't get it your question
 
10:43 AM
and you have it open, press file and click new notebook and then?
I meant student edition, professional etc
 
@Committingtoaname It perfectly works if I work in online mode
 
Huy
@Anastasiya-Romanova: IIRC, the current version is 10.0.1 and I'm wondering why you're using 9.0.0.0 instead.
 
Oh, I dunno that. Maybe student
 
I have version 9.0 personally, it is student edition
 
Huy
@Committingtoaname: I have a student's version as well, but always the most recent one, 10.0.1 atm.
 
10:45 AM
@Huy That's I get from my dad
 
Huy
Okay.
 
I can't actually find where to update it
 
Hmmm, what's a good program to draw 3D surfaces?
 
@Huy Should I buy the version 10 now?
 
Preinstalled "Grapher" on Mac OS is pretty good.
 
Huy
10:47 AM
@Committingtoaname: user.wolfram.com/portal I can download the most recent version there.
@Anastasiya-Romanova: I don't think it would be worth it but I was just wondering why you didn't use the most recent one.
 
@Huy I just asked from my dad I wanted Mathematica & he gave me this version
 
I am only allowed 9.0.1, it says that is my product version, oh well haha
 
@Chris'ssis I've posted my better asymptotic expansion and using it and computing the double sum for $n=10000$, I get the constant to 70 places.
 
@Huy So if I want an iPhone, it means I should buy the latest one?
 
Huy
@Anastasiya-Romanova: Depends on your needs. My new iPhone 6 will arrive tomorrow.
 
10:50 AM
Have you seen the bending?
It's terrible!
Apple have not responded yet...I'm not quite sure what they're going to do about it.
 
Probably not the right room, but, is it against the rules to ask for coding examples on math.se?
I can't seem to find them :/
 
@Huy Now you're becoming annoying
 
Huy
@GustavoMontano: I bought iPhone 4 despite the antennagate and never experienced comparable problems.
@Anastasiya-Romanova: Or you're just oversensitive.
 
I can swear that @Huy is a nice guy
5
 
Hopefully it isn't a global problem.
 
10:53 AM
@Huy: What do you mean by this one: "My new iPhone 6 will arrive tomorrow."? Are you trying to show off here or what?
 
@robjohn That job is crazy impressive. I would have never ever thought of that. :-)
 
No one asks your new iPhone 6
 
Huy
@GustavoMontano: Also, for now, Apple are actually replacing bent iPhones until they have figured out whether it is actually a problem on their side and what they will do about it.
@Anastasiya-Romanova: You brought up the iPhone, not me.
 
I would buy the iPhone 6. But a jailbroken iPhone is far superior to my needs.
 
But I didn't ask about your damn iPhone 6
 
10:54 AM
@Anastasiya-Romanova is mad.
When a girl is mad, you say sorry.
 
Huy
@GustavoMontano: You can't expect any company to just blindly announce "we will ship out new unbendable iPhones for everyone next week" - they have to inspect the problem closely.
 
Obviously xD.
 
@Chris'ssis I have computed the cosh sum in karvens answer... I need to find it.
 
Huy
@GustavoMontano: Similar problems also occured with different phones but the press never cared because it wasn't Apple. For example, check out this thread: forum.xda-developers.com/showthread.php?t=2287882&page=1
 
@robjohn Yeah, I remember that! I wanted to tell you that. Just let me know if you find it.
 
10:56 AM
@Chris'ssis before I read the rest of his answer, I want to see if I can compute the limit using that sum...
 
Hmmm, that is true. It is Apple. They have quite the reputation.
 
Huy
@Anastasiya-Romanova: Believe me, if I wanted to show off I wouldn't do it in an online chatroom where nobody will ever meet me anyways. No point in doing that. Just calm down. I didn't mean to offend you.
@Anastasiya-Romanova: Me bringing up my iPhone was meant in a humourous way but unfortunately you didn't take it that way.
I need to leave for lunch now. Have a nice day everyone!
 
Cya @Huy!
 
@robjohn I got 25 votes on that question, it seems people are in love with that. Indeed, it's an exceptionally nice question.
and a "Good question" badge
@robjohn you should pose questions more often :-)
 
@Chris'ssis I was going to ask that one since I did say "the interesting question is..." but you beat me to it.
 
11:08 AM
:D
 
@robjohn When you have the time, could you look at the proof of theorem (1) in the following answer? math.stackexchange.com/questions/884021/… I'm pretty sure it's not correct because it returns the wrong value for $s=1$. Did things go awry when he changed the order of integration?
 
@Analysis!
How're ya doing?
 
Great thank you @Kha!
How are you?
 
@RandomVariable I will have to look a bit later...
 
11:23 AM
@robjohn OK
 
@Chris'ssis I cannot find it. I wonder if it was just in chat...
@Chris'ssis it is $(2)$ in this answer
 
@robjohn Isn't $(2)$ the well-known partial fraction expansion of $\coth(x)$?
 
@Chris'ssis I guess it depends on what you call "well-known"
 
@Chris'ssis So you define "well-known" as being in the DLMF?
 
11:34 AM
@robjohn No, but I used that many times, and it's well-known I think.
 
In any case, that was the place I was thinking of
 
@robjohn OK
 
I knew it off the top of my head personally
(I didn't[I am joking])
 
@robjohn There is a confusion though. I thought you referred to this sum $$\sum_{m=1}^\infty\frac1{m(e^{2\pi m}-1)}$$
@robjohn I saw you computing it ... (I don't remember where)
 
@Chris'ssis Oh, yeah... I was just looking at that too...
 
11:44 AM
Is there a way to view older starred posts?
Nvm found
 

« first day (1513 days earlier)      last day (3515 days later) »